ChaseDream
搜索
返回列表 发新帖
查看: 1296|回复: 4
打印 上一主题 下一主题

GWD-05-Q38

[复制链接]
楼主
发表于 2007-12-14 18:52:00 | 只看该作者

GWD-05-Q38

Q38:

Three large companies and seven small companies currently manufacture a product with potential military applications.  If the government regulates the industry, it will institute a single set of manufacturing specifications to which all ten companies will have to adhere.  In this case, therefore, since none of the seven small companies can afford to convert their production lines to a new set of manufacturing specifications, only the three large companies will be able to remain in business.

 

Which of the following is an assumption on which the author’s argument relies?

 

  1. None of the three large companies will go out of business if the government does not regulate the manufacture of the product.
  2. It would cost more to convert the production lines of the small companies to a new set of manufacturing specifications than it would to convert the production lines of the large companies.
  3. Industry lobbyists will be unable to dissuade the government from regulating the industry.
  4. Assembly of the product produced according to government manufacturing specifications would be more complex than current assembly procedures.
  5. None of the seven small companies currently manufactures the product to a set of specifications that would match those the government would institute if the industry were to be regulated.

答案是E

C 为什么是错的?

沙发
发表于 2007-12-14 20:17:00 | 只看该作者

1  这是一道假设题,可以用无关词排除法,C中出现原文中没有的 lobbyists ,所以错了。

2   文中的结论是since none of the seven small companies can afford to convert their production lines to a new set of manufacturing specifications, only the three large companies will be able to remain in business.作者认为7个small companies 不存在是因为cannot afford .....specifications

E选项中加not后选项正好削弱结论,说这几个small companies would match those the government would institute if the industry were to be regulated.

E选项中加not后选项正好削弱结论,说这几个small companies would match those the government would institute if the industry were to be regulated.

E选项中加not后选项正好削弱结论,说这几个small companies would match those the government would institute if the industry were to be regulated.

E选项中加not后选项正好削弱结论,说这几个small companies would match those the government would institute if the industry were to be regulated.

板凳
 楼主| 发表于 2007-12-16 00:17:00 | 只看该作者
有无关词的,一定是错的吗?
地板
发表于 2007-12-16 21:45:00 | 只看该作者
对!逻辑题只有假设和结论两种题型可以用无关词排除法
5#
 楼主| 发表于 2007-12-17 00:40:00 | 只看该作者

请问:assumption 和 strengthen 两种题型有什么区别, 好像有时候是一样的。

您需要登录后才可以回帖 登录 | 立即注册

Mark一下! 看一下! 顶楼主! 感谢分享! 快速回复:

手机版|ChaseDream|GMT+8, 2024-9-28 23:37
京公网安备11010202008513号 京ICP证101109号 京ICP备12012021号

ChaseDream 论坛

© 2003-2023 ChaseDream.com. All Rights Reserved.

返回顶部